Audio Qbank – Q30 – A 69M with an MI

 

A 69-year-old male has an episode of crushing chest pain. ECG shows ST-segment elevation in leads II, III, and aVF. If his coronary anatomy bears left-dominant circulation, occlusion of which vessel is most likely in this patient?

  Left anterior descending artery
  Left circumflex artery
  Left main coronary artery
  Posterior descending artery
  Right main coronary artery

 

Cut-to-the-chase Answer (0:51):
HY USMLE Answer (4:03):